A Simple Proof Of Bell's Theorem

In summary: What it does not tell you is that this "communication" would necessarily involve the transmission of physical particles between the two polarizers. In other words, it does not tell you how to achieve instantaneous communication between particles at any distance.
  • #1
alt
Gold Member
222
0
I've been following up a lot regarding the fascinating stuff I read here.

I recently came across this page;

http://quantumtantra.com/bell2.html

I know that the author and the other material on that site would be considered new age crackpottery by most here at PF, but anyway, that page for me is a simple and lucid explanation, just within my level of undesrtanding of these things. Had it been a little more complex, I would not have understood it. So I'm gald I found it, though I have a few questions;

- What do others think of the explanation ?

- Has it been proven by repeated experiment/s, or is it pure theory / maths ?

- If it HAS been proven by experiments, to what extent if any, has it been put to practical use ? I mean, instantanious communication between particles at any distance .. WOW !

Any feedback would be appreciated. Thanks.
 
Physics news on Phys.org
  • #2
alt said:
I've been following up a lot regarding the fascinating stuff I read here.

I recently came across this page;

http://quantumtantra.com/bell2.html

I know that the author and the other material on that site would be considered new age crackpottery by most here at PF, but anyway, that page for me is a simple and lucid explanation, just within my level of undesrtanding of these things. Had it been a little more complex, I would not have understood it. So I'm gald I found it, though I have a few questions;

- What do others think of the explanation ?

- Has it been proven by repeated experiment/s, or is it pure theory / maths ?

- If it HAS been proven by experiments, to what extent if any, has it been put to practical use ? I mean, instantanious communication between particles at any distance .. WOW !

Any feedback would be appreciated. Thanks.

Nick Herbert's explanation is fine. Bell tests are regularly performed (often in undergraduate labs) and support the predictions of quantum mechanics. Local realism has been experimentally rejected. Despite this conclusion, you cannot use the mechanism for FTL communication. The "signal" is random, not much you can get out of that.

The only thing I object to is Nick's claim to have the shortest proof. I think this is shorter, the actual proof being 8 paragraphs. It is certainly written by a better looking person (that being moi).

http://drchinese.com/David/Bell_Theorem_Easy_Math.htm
 
  • #3
alt said:
I've been following up a lot regarding the fascinating stuff I read here.

I recently came across this page;

http://quantumtantra.com/bell2.html

I know that the author and the other material on that site would be considered new age crackpottery by most here at PF, but anyway, that page for me is a simple and lucid explanation, just within my level of undesrtanding of these things. Had it been a little more complex, I would not have understood it. So I'm gald I found it, though I have a few questions;

Yes, Nick Herbert is something of a crackpot. Nevertheless, his assessment of the meaning of Bell's theorem (ie., the meaning of violations of Bell inequalities) might well be the mainstream view (although nobody really knows because no polls or surveys of physicists' opinions on this exist).

alt said:
- What do others think of the explanation ?
Some, maybe most, think it's incomplete and misleading. This isn't Herbert's field. In fact, this field, the meaning of Bell's theorem, is the concern of an extremely small minority of working physicists.

alt said:
- Has it been proven by repeated experiment/s, or is it pure theory / maths ?
Has what been proven? That's the question. The opinion of many researchers in this field (of which Herbert is not one) is that violations of BIs do not imply nonlocality, or anything else about nature.

alt said:
- If it HAS been proven by experiments, to what extent if any, has it been put to practical use ?
If WHAT has been proven by experiments? Nonlocality? No. It's unverifiable and unfalsifiable. It's just not physics.

alt said:
I mean, instantaneous communication between particles at any distance .. WOW !
Wow indeed. But there's no evidence for it. And there can never be any evidence for it. It's a completely nonphysical concept, if it can be considered a concept at all. Think about it. Communication implies some sort of propagation or transmission between spatially separated objects, and instantaneous says that two events are happening at exactly the same time. So, 'instantaneous communication' is something of a nonsensical oxymoron. Isn't it?

alt said:
Any feedback would be appreciated. Thanks.

Ok. Having said the above, let me explain to you what Nick Herbert's simplification of Bell's theorem does and does not tell you.

What it does tell you is that if the light incident on the polarizers is modeled in terms of discrete and separable "instruction sets" or "coded messages", then it would be expected that the rate of coincidental detection would vary linearly as a function of the angular difference between the polarizer settings.

What it doesn't tell you is that it's been known for about 200 years that light doesn't behave in this way. The simplest way to illustrate this is to move one of the polarizers to the other side so that both polarizers are between the emitter and detector A or between the emitter and detector B. The results are exactly the same as when one polarizer is on side A and one polarizer is on side B. With both polarizers on one side, the rate of coincidental detection is the same as the rate of detection on the side that both polarizers are on.

Now, DrC might tell you that this is just a coincidence. DrC might even tell you that optical laws have nothing to do with optical Bell tests. But keep in mind that DrC is just a computer programmer of unknown competence. He's not a physicist. He's not an expert in optics, certainly not quantum optics. And, he's not an expert logician or mathematician. People who are experts in these fields have published papers showing that violations of BIs do not imply anything about nature. Who are you going to believe?

If you want to get at the truth of things, and be satisfied that you understand everything involved, then just ignore all the above and do your own research. Learn about classical and quantum optics. Read as much as you can of the relevant papers on Bell, GHZ, Hardy, etc. And come to your own conclusions. This should take you at least a couple of years unless you're ridiculously intelligent.

Then, hopefully, you can explain this stuff to me in a way that I can actually understand it.
 
  • #4
ThomasT said:
Some, maybe most, think it's incomplete and misleading.
Nonsense. You have shown in the past that you have little detailed understanding of the subject, and that you have an axe to grind against Bell's theorem which seems to be based on mere intuition that there's something fishy about it rather than being able to point to any specific flaw, but that doesn't justify fantasizing that "most" physicists would take your side!
ThomasT said:
Has what been proven? That's the question. The opinion of many researchers in this field (of which Herbert is not one) is that violations of BIs do not imply nonlocality, or anything else about nature.
Again, nonsense. The opinion of virtually all mainstream researchers is that violations of Bell inequalities (which are predicted theoretically by QM) imply that all local realist models can be ruled out (you will find almost no mainstream researchers who dispute this on a theoretical level, though a few might say the imperfections of experiments to date give rise to serious doubt about whether violations of Bell inequalities would still be seen with an ideal experimental setup). Who are the "many researchers in this field" you refer to? Can you point to peer-reviewed papers which dispute the basic theoretical conclusion that violations of Bell inequalities (under the experimental conditions given by Bell) are sufficient to rule out local realism?
ThomasT said:
If WHAT has been proven by experiments? Nonlocality? No. It's unverifiable and unfalsifiable. It's just not physics.
What has been proven theoretically is that local realism implies the Bell inequalities, and experiments show violations of Bell inequalities (although no experiment has been 100% ideal, the detection loophole and locality loophole have not both been simultaneously closed off by any experiment), so that's evidence that local realism is false. That's not the same as saying nonlocality is true, since you can dispense with "realism" (as in the Copenhagen interpretation) or you can dispense with other conditions assumed by local realism like the one that says each measurement has a single definite outcome (which isn't true in the many-worlds interpretation), but it's definitely "physics" to show theoretically that local realism --> Bell inequalities.
ThomasT said:
What it does tell you is that if the light incident on the polarizers is modeled in terms of discrete and separable "instruction sets" or "coded messages", then it would be expected that the rate of coincidental detection would vary linearly as a function of the angular difference between the polarizer settings.

What it doesn't tell you is that it's been known for about 200 years that light doesn't behave in this way.
What are you talking about when you say "200 years"? None of the traditional classical optics experiments involve situations where each experimental setting (like a choice of polarizer angle) gives one of two possible measurement results, which is the type of experiment that Bell's theorem is dealing with. Of course you could artificially design a classical experiment where a green light would go off if the intensity of light passing through the polarizer was above a certain threshold and a red light would go off if the intensity was below that threshold, but the intensity could still vary continuously, and this type of classical experiment would not give any violations of Bell inequalities.
ThomasT said:
The simplest way to illustrate this is to move one of the polarizers to the other side so that both polarizers are between the emitter and detector A or between the emitter and detector B.
What is that supposed to "illustrate"? Are you considering a classical experiment where the measuring apparatus is designed in such a way that each measurement gives one of two possible results? If so, then again, putting the polarizers on the same side will not give any violation of Bell inequalities--do you disagree?
ThomasT said:
The results are exactly the same as when one polarizer is on side A and one polarizer is on side B. With both polarizers on one side, the rate of coincidental detection is the same as the rate of detection on the side that both polarizers are on.

Now, DrC might tell you that this is just a coincidence.
Why should he have to do that, given that there will be no classical violation of Bell inequalities here?
ThomasT said:
People who are experts in these fields have published papers showing that violations of BIs do not imply anything about nature. Who are you going to believe?
Peer-reviewed papers that have been accepted as valid by mainstream physicists? If so, where?
 
  • #5
ThomasT said:
Some, maybe most, think it's incomplete and misleading.
JesseM said:
Nonsense. You have shown in the past that you have little detailed understanding of the subject, and that you have an axe to grind against Bell's theorem which seems to be based on mere intuition that there's something fishy about it rather than being able to point to any specific flaw, but that doesn't justify fantasizing that "most" physicists would take your side!
Nick Herbert's account is incomplete and misleading. Period.

I think that most physicists would say that nature is evolving according to local and discoverable underlying dynamics. Am I wrong?

ThomasT said:
Has what been proven? That's the question. The opinion of many researchers in this field (of which Herbert is not one) is that violations of BIs do not imply nonlocality, or anything else about nature.
JesseM said:
Again, nonsense. The opinion of virtually all mainstream researchers is that violations of Bell inequalities (which are predicted theoretically by QM) imply that all local realist models can be ruled out ...
Your reply doesn't address what I said. Are you deliberately trying to confuse people?

JesseM said:
... (you will find almost no mainstream researchers who dispute this on a theoretical level, though a few might say the imperfections of experiments to date give rise to serious doubt about whether violations of Bell inequalities would still be seen with an ideal experimental setup). Who are the "many researchers in this field" you refer to? Can you point to peer-reviewed papers which dispute the basic theoretical conclusion that violations of Bell inequalities (under the experimental conditions given by Bell) are sufficient to rule out local realism?
Experimental imperfections are not the problem. What's disputed is the logic involved in the development/derivation of Bell inequalities, and the subsequent interpretations thereof. I provided links to two papers that you can critique. So, critique them.

JesseM said:
What has been proven theoretically is that local realism implies the Bell inequalities ...
No. Not the totality of local realistic conceptions, but just the ridiculously simplistic ones associated with Herbert's "coded messages" or Mermin's (or whoever) "instruction sets". An alternative interpretation of the results is that what's been theoretically proven, and experimentally shown, is that light can't behave that way. But this comes as no surprise, because we already knew that it doesn't behave that way.

JesseM said:
... and experiments show violations of Bell inequalities (although no experiment has been 100% ideal, the detection loophole and locality loophole have not both been simultaneously closed off by any experiment), so that's evidence that local realism is false.
No, it isn't. You need to read the papers I recently linked to.

JesseM said:
That's not the same as saying nonlocality is true, since you can dispense with "realism" (as in the Copenhagen interpretation) or you can dispense with other conditions assumed by local realism like the one that says each measurement has a single definite outcome (which isn't true in the many-worlds interpretation), but it's definitely "physics" to show theoretically that local realism --> Bell inequalities.
Just read the papers and let me know what you think.

JesseM said:
What are you talking about when you say "200 years"? None of the traditional classical optics experiments involve situations where each experimental setting (like a choice of polarizer angle) gives one of two possible measurement results, which is the type of experiment that Bell's theorem is dealing with. Of course you could artificially design a classical experiment where a green light would go off if the intensity of light passing through the polarizer was above a certain threshold and a red light would go off if the intensity was below that threshold, but the intensity could still vary continuously, and this type of classical experiment would not give any violations of Bell inequalities.
Quantum polarimeters produce results according to the same optical law that classical polarimeters do. Don't obfuscate this. Herbert's account of Bell's theorem is quite on the mark, and it shows quite simply and easily a very profound problem with Bell's assumptions. We're expected to suppose that light is going to behave in Bell experiments in a way that it has never before been observed to behave. And, if it doesn't behave in this strange way, then we're supposed to conclude that nature is either nonlocal or that nature doesn't exist. It's absolutely absurd what you and DrC are promoting as 'science' to the public.

Read the papers by Hess, et al., then I'll have some more for you to look at.

ThomasT said:
What is that supposed to "illustrate"? Are you considering a classical experiment where the measuring apparatus is designed in such a way that each measurement gives one of two possible results? If so, then again, putting the polarizers on the same side will not give any violation of Bell inequalities--do you disagree?
Putting the polarizers on the same side (in an optical Bell test setup w/entangled photons) will give exactly the same results as having one on each side -- ie., a violation of BI. So, in the altered setup, you have a quantum polariscope on one side which determines the rate of coincidental detection. There's, presumably, nothing nonlocal going on in this setup. So, why, when we put one of the polarizers back on the other side, and get the same results, should we assume that there's something nonlocal going on in this, the original, setup? Or, consider an OPDC setup where a slight adjustment of a wave plate produces entanglement stats. Are we to suppose that this slight adjustment suddenly conjured up some new (and apparently arbitrarily varying) underlying ftl communicative medium or field (it can't be the EM field), or is there maybe a simpler explanation, maybe having something to do with, say, phases, or whatever?

JesseM said:
Peer-reviewed papers that have been accepted as valid by mainstream physicists? If so, where?
Are you a peer of people like Hess, De Raedt, Sica, Dalton, Christian etc.? If so, then you can review and critique their papers for the benefit of the folks here at PF, and, who knows, maybe at least one of them will visit PF to discuss his/her paper with you.

Start with the papers that I linked to please. And please, no more nitpicky obfuscating responses on stuff that I've written. I'm just a curious amateur. I don't know what your qualifications are, but the people who wrote the papers I want you to critique are the real deal.
 
  • #6
DrChinese said:
Nick Herbert's explanation is fine. Bell tests are regularly performed (often in undergraduate labs) and support the predictions of quantum mechanics. Local realism has been experimentally rejected. Despite this conclusion, you cannot use the mechanism for FTL communication. The "signal" is random, not much you can get out of that.

The only thing I object to is Nick's claim to have the shortest proof. I think this is shorter, the actual proof being 8 paragraphs. It is certainly written by a better looking person (that being moi).

http://drchinese.com/David/Bell_Theorem_Easy_Math.htm

Thanks, DrChinese. This is all very new and interesting to me. I have read your link and it is very informative.

ThomasT and JesseM, thank you also for the replies.

Even though you guys are at odds on this, I am really interested in what you're each saying, and avidly taking it all in - to the best of my QP abilities anyway (which are pretty much zilch).
 
  • #7
ThomasT said:
But keep in mind that DrC is just a computer programmer of unknown competence.

I don't know about that one! Tens of thousands of copies of my commercial software have been sold over the years.

Also: Besides being charming and handsome, I am also modest. So don't dismiss me!
 
  • #8
alt said:
Thanks, DrChinese. This is all very new and interesting to me. I have read your link and it is very informative.

You are welcome.

As a side note, I would recommend disregarding everything ThomasT says. Either that, or read it along with the morning comics. :biggrin:
 
  • #9
DrChinese said:
You are welcome.

As a side note, I would recommend disregarding everything ThomasT says. Either that, or read it along with the morning comics. :biggrin:

:rofl: to this and your last post, you're vying with DaveC for the humor award!

Fun aside, thanks for the link to the other proof, I enjoyed that a great deal.
 
  • #10
DrChinese said:
You are welcome.

As a side note, I would recommend disregarding everything ThomasT says. Either that, or read it along with the morning comics. :biggrin:

Lol, thanks.

Antinomy itself is often instructive !
 
  • #11
ThomasT said:
I think that most physicists would say that nature is evolving according to local and discoverable underlying dynamics. Am I wrong?
If you define "local" in the sense of local realism (as opposed to some weaker sense like it being impossible for experimenters to exploit the laws of nature to send FTL signals), then yes, you are wrong.
ThomasT said:
Your reply doesn't address what I said. Are you deliberately trying to confuse people?
It directly addresses what you said when you claimed "The opinion of many researchers in this field (of which Herbert is not one) is that violations of BIs do not imply nonlocality, or anything else about nature." Asking whether or not the universe's fundamental laws are local realist ones is certainly a question "about nature", and the opinion of virtually all mainstream researchers is that Bell's theorem proves on a theoretical level that violations of Bell inequalities imply local realism must be false.
ThomasT said:
What's disputed is the logic involved in the development/derivation of Bell inequalities, and the subsequent interpretations thereof. I provided links to two papers that you can critique. So, critique them.
Huh? You have provided no links!
JesseM said:
What has been proven theoretically is that local realism implies the Bell inequalities ...
ThomasT said:
No. Not the totality of local realistic conceptions, but just the ridiculously simplistic ones associated with Herbert's "coded messages" or Mermin's (or whoever) "instruction sets".
You are simply wrong here (and given your own lack of knowledge of physics, it's ridiculous that you act so confident), the mainstream view is that the "totality of local realistic conceptions" have indeed been shown to be incompatible with violations of Bell inequalities. For example, local realist theories where the measurement outcome is determined by a combination of hidden variables associated with the particle (which may change as it travels) and hidden variables associated with the region of the experimental apparatus are ruled out too by BI violations. BI violations are incompatible with any theory that is "local realist" in the sense that A) all physical facts about a region of spacetime can be reduced to a collection of facts about local variables associated with individual points in spacetime, and B) the value of each local variable is only causally influenced by variables in its past light cone.
ThomasT said:
Quantum polarimeters produce results according to the same optical law that classical polarimeters do. Don't obfuscate this.
Don't rudely accuse me of obfuscation if you can't point out where I am wrong. Do you disagree that it would be impossible to replicate the violations of BI predicted by QM if we were doing purely classical optics experiments? (note that Maxwell's laws of classical electromagnetism, from which classical optics can be derived, are a fine example of local realist laws!) If quantum polarimeters produce BI violations and classical polarimeters never do, that would suggest they are not following the "same law".

Perhaps by "same law" you just mean that the classical Malus' law for polarized light and the law for entangled particles both involve a cos^2? The problem is that although the equation can be written in a similar form for both laws, the physical meaning of the symbols is completely different, so from a physical perspective they cannot be called the "same". If you write cos^2(a-b) in the classical context a would be the polarization angle of the light, b would be the angle of a single polarizer, and cos^2 would be giving the reduction in intensity of the light as it passes through the polarizer; but if you write cos^2(a-b) in the quantum context, a and b would both be polarizer angles, there would be no term for the polarization of the light, and cos^2 would be giving the probability that both photons give the same binary result (both passing through their polarizers, or neither).
ThomasT said:
Herbert's account of Bell's theorem is quite on the mark, and it shows quite simply and easily a very profound problem with Bell's assumptions. We're expected to suppose that light is going to behave in Bell experiments in a way that it has never before been observed to behave.
What do you mean "never been observed to behave"? Again, in a classical optics experiment with a setup meeting Bell's conditions (spacelike separation between measurements, each measurement giving one of two possible binary results, etc.) you'd always observe light obeying the Bell inequalities, do you disagree?
ThomasT said:
It's absolutely absurd what you and DrC are promoting as 'science' to the public.
It's absolutely absurd that you, obviously knowing very little about the literature and having only your own intuitions telling you there's something wrong with Bell's theorem (since you have never been able to point to a specific flaw or explain how a local realist model could lead to BI violations), confidently make the completely false claim that lots of physicists think Bell's theorem is flawed based on two papers you've found out of the vast literature on the subject (which you probably don't understand in any detail yourself). Confirmation bias is an amazing thing! But despite your wishful thinking, rest assured the notion that Bell's proof is invalid is a thoroughly fringe notion, and DrC and myself are just expressing the mainstream view which virtually all physicists who have studied the subject would agree with.
ThomasT said:
Read the papers by Hess, et al., then I'll have some more for you to look at.
Are you referring to the paper that Gill, Weihs, Zeilinger, and Zukowski claim to refute here (full paper on arxiv.org here), as does Myrvold http://webcache.googleusercontent.com/search?q=cache:9Ce5Q2whYP8J:philsci-archive.pitt.edu/archive/00001090/00/Myrvold.doc+Hess+Bell's+theorem&cd=5&hl=en&ct=clnk&gl=us]here[/PLAIN] [Broken] and Mermin here and here? Hess' argument hasn't achieved any acceptance among quantum physicists, and in any case Hess' argument seems to be based on the notion that Bell neglected to consider time-dependent hidden variables, but the version of Bell's theorem which let's the hidden variables stand for all physical facts in the past light cones of the measurement at some specific time before the measurement (but after the last time the two light cones overlap), which is the version I have been discussing all along (for example, in posts 61 and 62 here...Bell also defined the hidden variables this way on p. 242 of Speakable and Unspeakable in Quantum Mechanics here), is perfectly consistent with the possibility of time-dependent hidden variables.
ThomasT said:
What it doesn't tell you is that it's been known for about 200 years that light doesn't behave in this way. The simplest way to illustrate this is to move one of the polarizers to the other side so that both polarizers are between the emitter and detector A or between the emitter and detector B.
JesseM said:
What is that supposed to "illustrate"? Are you considering a classical experiment where the measuring apparatus is designed in such a way that each measurement gives one of two possible results? If so, then again, putting the polarizers on the same side will not give any violation of Bell inequalities--do you disagree?
ThomasT said:
Putting the polarizers on the same side (in an optical Bell test setup w/entangled photons) will give exactly the same results as having one on each side -- ie., a violation of BI.
I was talking about a classical optical experiment, not a quantum experiment with entangled photons, as I made clear above. And if you were talking about a quantum experiment, then using that to "illustrate" your comment that "it's been known for about 200 years that light doesn't behave this way" is completely incoherent, since no one was performing experiments with entangled photons until the 20th century.
ThomasT said:
So, in the altered setup, you have a quantum polariscope on one side which determines the rate of coincidental detection. There's, presumably, nothing nonlocal going on in this setup.
I'm not clear on what setup you're assuming in the quantum version. Does each photon actually travel through two polarizers in succession before reaching the polariscope, or do the two entangled photons go in the same general direction but have slightly different paths so one only passes through/is blocked by polarizer #1 while the other only passes through/is blocked by polarizer #2?
ThomasT said:
So, why, when we put one of the polarizers back on the other side, and get the same results, should we assume that there's something nonlocal going on in this, the original, setup?
What "same results" are you talking about? To get a violation of BI you have to have multiple possible settings for the two polarizers, so you can sometimes observe the coincidence rate when both polarizers have the same angle but sometimes observe the coincidence rate when the polarizers are set to different angles. If we give the polarizers the possible angles 0, 60, and 120 (so that whenever the two polarizers have different angles a and b, cos^2(a-b) is always 0.25) when they are on the "same side" do we still have some trials where each member of the pair goes through a polarizer with a different angle? If so I have no idea why you'd say "There's, presumably, nothing nonlocal going on in this setup"--although it would be possible in this case for a local realist theory to replicate the statistics seen in QM, it would only be by exploiting the locality loophole, meaning there is some kind of "communication" going on between the particles where the polarizer angle encountered by one affects the behavior of the other (if on the other hand the particles have no causal influence on one another after leaving the source, then according to local realism the BI shouldn't be violated). If you then put the polarizers on opposite sides so that there's a spacelike separation between each photon encountering its polarizer, then either the communication has to break down (so the statistics change and you no longer see BI violations) or the "communication" will now have to work in an FTL manner.
ThomasT said:
Or, consider an OPDC setup where a slight adjustment of a wave plate produces entanglement stats. Are we to suppose that this slight adjustment suddenly conjured up some new (and apparently arbitrarily varying) underlying ftl communicative medium or field (it can't be the EM field)
Why "new"? I think in Bohmian mechanics the nonlocal quantum potential would always be guiding entangled particles, regardless of the statistics in the experiment, and doing so according to the same equation at all times. And as I've told you many times before, nonlocality is not the only alternative to local realism, one can also take options like the many-worlds interpretation.
JesseM said:
Peer-reviewed papers that have been accepted as valid by mainstream physicists? If so, where?
ThomasT said:
Are you a peer of people like Hess, De Raedt, Sica, Dalton, Christian etc.?
The whole concept of "peer-reviewed" is that professional physicists do the reviewing, which gives the rest of us some sense of whether the paper is likely to have a coherent argument. If you want to continue to dispute my point that the mainstream view is that Bell's theorem is valid, you should be able to point to peer-reviewed papers, as opposed to any ol' paper someone uploaded to arxiv.org or whatever. You haven't actually linked to papers by these authors so I don't know if they're peer-reviewed (and I also don't know whether they are actually disputing Bell's theorem on a theoretical level or pointing to other issues like the possibility of a local realist theory that exploits experimental loopholes and would obey the BI in an ideal experiment, as in the Da Raedt model DrChinese was discussing here)
ThomasT said:
Start with the papers that I linked to please. And please, no more nitpicky obfuscating responses on stuff that I've written. I'm just a curious amateur. I don't know what your qualifications are, but the people who wrote the papers I want you to critique are the real deal.
They may be "the real deal" in the sense of professionals, but if they question Bell's theorem on a theoretical level (and I don't know if the others besides Hess do), then they represent a decidedly fringe view which almost all mainstream physicists would disagree with. Please don't misrepresent a fringe view as being a commonplace opinion among physicists if you only have a few cherrypicked papers to back it up; these forums are designed to teach mainstream physics, so fringe views need to be flagged as such to avoid confusing readers.
 
Last edited by a moderator:
  • #12
nismaratwork said:
:rofl: to this and your last post, you're vying with DaveC for the humor award!

I quack myself up. :smile:
 
  • #13
alt said:
ThomasT and JesseM, thank you also for the replies.
You're most welcome alt. Did you actually agree with anything I said?

alt said:
Antinomy itself is often instructive !
It seems to be pretty much how they roll here at PF.

DrChinese said:
I don't know about that one! Tens of thousands of copies of my commercial software have been sold over the years.
I hereby retract my statement. Better yet just change "unknown" to "questionable". Just kidding. I'm sure you're quite good at what you do.

DrChinese said:
Also: Besides being charming and handsome, I am also modest. So don't dismiss me!
How could anyone possibly dismiss you? I was just trying to spice up the antinomy, whatever that means, in between my normal duties as intellectual swindler #2 (I'm going for the #1 spot).

DrChinese said:
As a side note, I would recommend disregarding everything ThomasT says. Either that, or read it along with the morning comics.

nismaratwork said:
:rofl: to this and your last post, you're vying with DaveC for the humor award!
Ok, it wasn't THAT funny.
 
  • #14
ThomasT said:
I think that most physicists would say that nature is evolving according to local and discoverable underlying dynamics. Am I wrong?
JesseM said:
If you define "local" in the sense of local realism (as opposed to some weaker sense like it being impossible for experimenters to exploit the laws of nature to send FTL signals), then yes, you are wrong.
So, you think that most physicists would say that nature is not evolving according to local and discoverable underlying dynamics?

ThomasT said:
Asking whether or not the universe's fundamental laws are local realist ones is certainly a question "about nature", and the opinion of virtually all mainstream researchers is that Bell's theorem proves on a theoretical level that violations of Bell inequalities imply local realism must be false.
I don't think that's the case. Anyway, here's those papers that I forgot to post here. The authors disagree with your assessment of Bell.

Possible Experience: from Boole to Bell
http://arxiv.org/PS_cache/arxiv/pdf/...907.0767v2.pdf [Broken]
Published in: EPL, 87 (2009) 60007

Extended Boole-Bell inequalities applicable to quantum theory
http://arxiv.org/PS_cache/arxiv/pdf/...901.2546v2.pdf [Broken]

The second paper, still in the works, on the extended Boole-Bell inequalities, provides a detailed account of why BIs are violated and why their violation doesn't imply nonlocality in nature.

ThomasT said:
Quantum polarimeters produce results according to the same optical law that classical polarimeters do.
JesseM said:
If quantum polarimeters produce BI violations and classical polarimeters never do, that would suggest they are not following the "same law".
First, just consider quantum and classical polarimeters without reference to BIs or optical Bell tests. They both produce results in accord with Malus Law.

Now consider, say, an Aspect or F and C optical Bell test setup where you have an emitter of entangled photons between two polarizers, a and b, between two detectors, A and B, respectively. Take the polarizer, a, and place it between the emitter and the polarizer, b. So, on the left side there is just a detector, A, and on the right side there are polarizers, a and b, between the emitter and detector B. The right side is now a polarimeter. It produces results in accord with Malus Law, as all polarimeters do. And the coincidence rate, ie. F(AB), is the same as with the original setup, .5 cos^2 |a-b|, in the ideal. But Herbert, vis Bell, requires that the polarimeter produce a linear relationship between |a-b| and rate of detection. This is why I said that light was being required by Herbert, vis Bell, to behave in a way contrary to what thousands of polarimetric experiments have shown.

JesseM said:
... rest assured the notion that Bell's proof is invalid is a thoroughly fringe notion ...
Nobody's arguing about Bell's mathematical proof, or the validity of experiments in which BIs are violated. What's being contested is the interpretation of BI violation, leading to a more appropriate phrasing of Bell's theorem.


So, JesseM, thanks for bearing with me. I'm hoping that we might go through at least one of the papers I linked to. It will be very instructional for me, as well as probably for a lot of noncontributing readers of this thread. Maybe just take a page every few days or so and post your comments. Take it slow, because, you're right, at the moment I don't fully understand them.
 
Last edited by a moderator:
  • #15
ThomasT said:
You're most welcome alt. Did you actually agree with anything I said?

Hi ThomasT;

Yes, I read what you said. On another post, you also said "I'm just a curious amateur"

Well, if you are an amatuer, I'm a babe in the wilderness.

I'm going to keep reading 'round here, and see if I can understand (let alone agree or disagree with) anything.

It's fun though :-)
 
  • #16
ThomasT said:
So, you think that most physicists would say that nature is not evolving according to local and discoverable underlying dynamics?
Yes, virtually all of them would say this. Please stop trying to use a few cherrypicked papers where the authors disagree with this conclusion to suggest otherwise. If you want to get a sense of what most physicists think rather than just confirm your own biases, try looking at textbooks which discuss Bell's theorem (you could go to google books and entering the keywords "bell's theorem" and "inequality"), you'll notice that all the textbooks written by physicists just present it as a straight fact that Bell's theorem shows QM is incompatible with local realism (or local causality or theories of local hidden variables, the exact wording may vary), they don't suggest any controversy about this conclusion.
JesseM said:
Asking whether or not the universe's fundamental laws are local realist ones is certainly a question "about nature", and the opinion of virtually all mainstream researchers is that Bell's theorem proves on a theoretical level that violations of Bell inequalities imply local realism must be false.
ThomasT said:
I don't think that's the case.
Well it is, and as an amateur in the subject you can have little rational basis for believing otherwise--finding a few fringe authors who disagree doesn't qualify as a good basis. Again, try the experiment of looking in textbooks--would you disagree that if physics textbooks all present some theoretical conclusion as a fact, then it's a safe bet that this conclusion is one that the majority of physicists would agree with?
ThomasT said:
Anyway, here's those papers that I forgot to post here. The authors disagree with your assessment of Bell.

Possible Experience: from Boole to Bell
http://arxiv.org/PS_cache/arxiv/pdf/...907.0767v2.pdf [Broken]
Published in: EPL, 87 (2009) 60007

Extended Boole-Bell inequalities applicable to quantum theory
http://arxiv.org/PS_cache/arxiv/pdf/...901.2546v2.pdf [Broken]
Your URLs are messed up, should be here and here. The first deals specifically with the Leggett-Garg inequality which is somewhat different from other Bell-type inequalites as it requires the additional assumption that in two successive measurements on the same system (with a timelike, not a spacelike separation), the first measurement will not influence the probability distribution for possible results on the second. And their example involving doctors and patients simply does not conform to the experimental conditions assumed by the inequality, see my discussion of this paper with billschnieder in post 941 and post 961 on this thread (I got sidetracked from continuing that discussion with bill but I intend to follow up on his last comment to me soon).

The first paper did appear in a peer-reviewed publication, the epl journal, perhaps because it didn't make the strong claim that the example they provided was actually a local realist violation of the Leggett-Garg inequality or any other Bellian inequality, you can interpret the paper as just being about an analogous inequality that fails to hold because the conditions under which the data is collected and indexed are different from those assumed in the derivation of Leggett-Garg (as well as the conditions assumed in the derivation of Boole's original inequality). The second paper, on the other hand, doesn't appear to have been published anywhere outside of arxiv.org (see google scholar search here), which doesn't require peer-review. I haven't read through it in detail, but skimming it the argument appears similar to the first paper, in that they are considering situations where the procedure by which data is collected and indexed does not conform to the conditions stipulated by Bell inequalities (or the conditions needed for a derivation of Boole's own inequality). In fact they mention the exact same example involving doctors and patients from the earlier paper on pp. 25-27 of this second paper. They do claim that it is possible to explicitly violate Bell's theorem with locally causal models on p. 28:
We have shown in a series of papers42,43,47,48,59 that it is
possible to construct models, that is algorithms, that are lo-
cally causal in Einstein’s sense, generate the data set Eq. (126)
and reproduce exactly the correlation that is characteristic for
a quantum system in the singlet state. These algorithms can
be viewed as concrete realizations of Fine’s synchronization
model8. According to Bell’s theorem, such models do not ex-
ist.data set leads to a violation of Boole’s inequalities, the
only conclusion that one can draw is that the data set
does not satisfy the conditions necessary to prove the
Boole inequalities, namely that three data sets of pairs
can be extracted from a single data set of triples (see
Section II).
...but the details of these models aren't provided in this paper, so there's no way to verify if the models actually conform to all the necessary conditions for Bell's proof. Back in post 62 of the thread 'Bell Theorem' I once suggested a test of any claim that a local model can replicate BI violations, by simulating the model using a set of computers which are cut off from communicating with one another at some time prior to the experimenter's choice of what detector setting to use (analogous to the fact that in Bell's proof, the past light cones of the two measurements stop overlapping at some time prior to the choice of detector setting for each measurement--each computer's internal state at a given time can be simulating an arbitrary number of local variables in the past light cone of a measurement). I feel totally confident that the authors of the paper would not be able to come up with an algorithm that would allow for BI violations in this scenario (which can stand for any of the inequalities proposed by Bell for measurements on pairs of particles at a spacelike separation, although it does not stand for the Leggett-Garg inequality where the two measurements have a timelike separation):
If anyone proposes that a local hidden variables theory can explain the results of these experiments, there's no reason that such a theory could not be simulated in the setup I described, where a middle computer M can send signals to two different computers A and B until some time t when the computers are disconnected, and some time after t the experimenters (real or simulated) make choices about which orientation to use for the simulated detector (if the experimenters are real people interacting with the simulation they could make this choice by deciding whether to type 1, 2, or 3 on the keyboard, for example), and each computer A and B must return a measurement result.
(note that in some Bell inequalities it's assumed that there are 2 possible detector settings for each experimenter rather than 3, in which case each experimenter could only choose to enter 1 or 2 on the keyboard of their computer on each trial and the computer would have to return a measurement result).

If you have followed the authors' argument in this second paper and think they show it would be possible to get a BI violation in this type of simulation, please explain. Likewise if you think they show it would be possible to get a violation of a BI in a scenario that matches all the observable experimental conditions stipulated in the derivation of the inequality, please explain. But if you haven't really followed the argument of the paper, and are just citing a paper you don't understand to support your claim that there is widespread disagreement among physicists about whether Bell's theorem showed QM was incompatible with all local realist theories, then one can show the absurdity of this claim just by noting that you're pointing to a non-peer-reviewed paper uploaded to arxiv.org which has led to no real reactions from the physics community (in the form of other physicists citing the paper--only four papers have cited it so far, two by De Raedt), and that physics textbooks which mention Bell's theorem uniformly present it as a valid demonstration that QM is incompatible with local models.
ThomasT said:
First, just consider quantum and classical polarimeters without reference to BIs or optical Bell tests. They both produce results in accord with Malus Law.
Malus' law is a classical law dealing with classical electromagnetic waves of known polarization, and the reduction in intensity of the wave when it passes through a polarizer. Since the reduction in intensity of a light beam passing through a polarizer is in QM equal to the fraction of photons that make it through the polarizer, I think it would be reasonable to say there is a quantum version of Malus' law in a situation where a set of photons are all in the same known polarization eigenstate at some angle (say, because they already passed through a first polarizer at that angle) and we want to know the fraction of them that will make it through a polarizer at a different angle (so in this scenario each photon encounters two polarizers in succession). However, if you're talking about an experiment involving pairs of entangled photons which each pass/don't pass through a single polarizer, it is meaningless to say that "Malus' law" predicts anything about this situation, since Malus' law only applies when you know the polarization of the light, and we don't know anything about the polarization of each pair when they are generated (and after they pass through polarizers, the entanglement between their polarizations is broken). Do you disagree?
ThomasT said:
Now consider, say, an Aspect or F and C optical Bell test setup where you have an emitter of entangled photons between two polarizers, a and b, between two detectors, A and B, respectively. Take the polarizer, a, and place it between the emitter and the polarizer, b. So, on the left side there is just a detector, A, and on the right side there are polarizers, a and b, between the emitter and detector B. The right side is now a polarimeter. It produces results in accord with Malus Law, as all polarimeters do. And the coincidence rate, ie. F(AB), is the same as with the original setup, .5 cos^2 |a-b|, in the ideal.
Why do you have an 0.5 in front of cos^2 (a-b)? I'm not sure of the exact "original setup" you're thinking of, but if the photons are entangled in such a way that identical measurements on each will yield the same results with probability 1, then if both polarizers are at the same angle so a-b=0, then the probability of getting the same result (either both passing through the polarizers, or both being reflected) should be cos^2(0) = 1, not 0.5 cos^2(0) = 0.5.

Assuming you should have written cos^2 (a-b) for the original setup, your altered setup above will not give the same results. After all, given perfectly efficient detection, on the left side every photon sent out by the emitter should be detected at A, since there are no polarizers to reflect photons sent to the left. On the right side, even if a and b are at the same angle so cos^2(a-b)=1, it is quite possible for some of the entangled photons to be reflected by polarizers at that angle rather than passing through, in which case the coincidence rate in this setup is not cos^2 (a-b).
ThomasT said:
But Herbert, vis Bell, requires that the polarimeter produce a linear relationship between |a-b| and rate of detection.
Presumably Herbert/Bell would not require this in any arbitrary setup like the one you describe above, only in the particular setup assumed in the derivation of the Bell inequalities, where a represents a detector setting in the region of the measurement at A and b represents a detector setting in the region of the measurement of B, with a spacelike separation between the two random choices of detector settings.
ThomasT said:
This is why I said that light was being required by Herbert, vis Bell, to behave in a way contrary to what thousands of polarimetric experiments have shown.
Your statistics don't actually seem to work as noted earlier, and in any case this is just a strawman if "Herbert, vis Bell" (can you point to what particular statement of Herbert you are talking about?) was only talking about such a linear relationship in the type of experiment assumed by Bell where there was a spacelike separation between the random choice of setting a and the random choice of setting b.
JesseM said:
... rest assured the notion that Bell's proof is invalid is a thoroughly fringe notion ...
ThomasT said:
Nobody's arguing about Bell's mathematical proof, or the validity of experiments in which BIs are violated. What's being contested is the interpretation of BI violation, leading to a more appropriate phrasing of Bell's theorem.
Whe I said "the notion that Bell's proof is invalid", I was talking about the notion that Bell's proof does not correctly show (as it purports to) that all local realist models would produce results satisfying Bell inequalities in the experimental setup Bell was assuming. The idea that Bell's proof is invalid in this broad sense is indeed a "thoroughly fringe notion", and the type of textbook search I suggested (or a representative search of the literature, rather than a search for cherrypicked examples of papers by the tiny number of physicists who disagree) will show this. Again, if you want to have a discussion about some paper that questions Bell's proof I (or others like DrChinese) can do that, but please stop trying to present disagreement with Bell as something that is common among physicists when you have nothing but wishful thinking to back it up.
ThomasT said:
So, JesseM, thanks for bearing with me. I'm hoping that we might go through at least one of the papers I linked to. It will be very instructional for me, as well as probably for a lot of noncontributing readers of this thread. Maybe just take a page every few days or so and post your comments. Take it slow, because, you're right, at the moment I don't fully understand them.
If there is some specific thing you have a question about we can discuss it, but I don't really feel like going through a detailed analysis of everything in those papers. You may find my comments about the first paper in the earlier discussion with billschnieder that I linked to helpful, and soon I plan to post a little more on the subject in response to bill's last post to me.
 
Last edited by a moderator:
  • #17
Now this is interesting reading; like alt I feel I'm learning from reading this discussion. Back on track, life is good.
 
  • #18
Paraphrased from Nick Herbert's book Quantum Reality, the following (borrowed from the OP's http://quantumtantra.com/bell2.html" [Broken]) is the best (simplest) account I have found to date for the Bell inequality (SPOT stands for Single Photon Orientation Tester):

Imagine that each random sequence that comes out of the SPOT detectors is a coded message. When both SPOT detectors are aligned, these messages are exactly the same. When the detectors are misaligned, "errors" are generated and the sequences contain a certain number of mismatches. How these "errors" might be generated is the gist of this proof.

Step One: Start by aligning both SPOT detectors. No errors are observed.

Step Two: Tilt the A detector till errors reach 25%. This occurs at a mutual misalignment of 30 degrees.

Step Three: Return A detector to its original position (100% match). Now tilt the B detector in the opposite direction till errors reach 25%. This occurs at a mutual misalignment of -30 degrees.

Step Four: Return B detector to its original position (100% match). Now tilt detector A by +30 degrees and detector B by -30 degrees so that the combined angle between them is 60 degrees.

What is now the expected mismatch between the two binary code sequences?

We assume, following John Bell's lead, that REALITY IS LOCAL.

Assuming a local reality means that, for each A photon, whatever hidden mechanism determines the output of Miss A's SPOT detector, the operation of that mechanism cannot depend on the setting of Mr B's distant detector. In other words, in a local world, any changes that occur in Miss A's coded message when she rotates her SPOT detector are caused by her actions alone.

And the same goes for Mr B. The locality assumption means that any changes that appear in the coded sequence B when Mr B rotates his SPOT detector are caused only by his actions and have nothing to do with how Miss A decided to rotate her SPOT detector.

So with this restriction in place (the assumption that reality is local), let's calculate the expected mismatch at 60 degrees.

Starting with two completely identical binary messages, if A's 30 degree turn introduces a 25% mismatch and B's 30 degree turn introduces a 25% mismatch, then the total mismatch (when both are turned) can be at most 50%. In fact the mismatch should be less than 50% because if the two errors happen to occur on the same photon, a mismatch is converted to a match.

Thus, simple arithmetic and the assumption that Reality is Local leads one to confidently predict that the code mismatch at 60 degrees must be less than 50%.

However both theory and experiment show that the mismatch at 60 degrees is 75%. The code mismatch is 25% greater than can be accounted for by independent error generation in each detector.

Therefore the locality assumption is false. Reality must be non-local.

Just beautiful.
 
Last edited by a moderator:
  • #19
ThomasT said:
So, you think that most physicists would say that nature is not evolving according to local and discoverable underlying dynamics?[
JesseM said:
Yes, virtually all of them would say this.
I think that you're wrong about this. So, unless you have some survey or poll of all physicists to back up your claim, then I guess that we can just disagree about this

JesseM said:
Why do you have an 0.5 in front of cos^2 (a-b)?
Why are you even asking this? This is what I mean by obfuscation. Obviously, if you have a source emitting randomly polarized photons, then if you place a polarizer between the emitter and the detector, then the coeffiicient of transmission with the polarizer is 1/2 the photon flux without the polarizer.

JesseM said:
I'm not sure of the exact "original setup" you're thinking of ...
It was clear enough.

JesseM said:
... but if the photons are entangled in such a way that identical measurements on each will yield the same results with probability 1, then if both polarizers are at the same angle so a-b=0, then the probability of getting the same result (either both passing through the polarizers, or both being reflected) should be cos^2(0) = 1, not 0.5 cos^2(0) = 0.5.
Don't be ridiculous. And unnecessarily obfuscating. The addition of a single polarizer between the emitter and the detector cuts the photon flux by 1/2.

JesseM said:
Assuming you should have written cos^2 (a-b) for the original setup, your altered setup above will not give the same results.
The prediction for the original setup will be .5cos^2|a-b| in the ideal. The altered setup, the polariscopic setup, will give the same results.

ThomasT said:
But Herbert, vis Bell, requires that the polarimeter produce a linear relationship between |a-b| and rate of detection.

ThomasT said:
This is why I said that light was being required by Herbert, vis Bell, to behave in a way contrary to what thousands of polarimetric experiments have shown.

Here's what you've failed to address so far. Bell and Herbert say that the number of mismatches at 30 degrees (a-b) plus the number of mismatches at 30 degrees should be less than or equal to the number of mismatches at 60 degrees. But, according to Malus Law the number of mismatches at 60 degrees should be greater than the number of mismatches at 30 degrees + the number of mismatches at 30 degrees. So, what you and Bell and Herbert are saying is contrary to 200 years of applied optics.

Of course, there's the possibility that OPTICAL Bell tests have nothing to do with OPTICS (per DrChinese et al.), but I think that that's just clutching at straws.

JesseM said:
If there is some specific thing you have a question about we can discuss it, but I don't really feel like going through a detailed analysis of everything in those papers.
Nor do I. In fact, I'm quite saturated with this topic for now and am quite amenable to letting it go for a while. I thank you for your detailed responses. And perhaps we will discuss this in future threads.
 
  • #20
Loren Booda said:
Paraphrased from Nick Herbert's book Quantum Reality, the following (borrowed from the OP's http://quantumtantra.com/bell2.html" [Broken]) is the best (simplest) account I have found to date for the Bell inequality (SPOT stands for Single Photon Orientation Tester):

...

Just beautiful.

Thanks for sharing that! You are right, it is simple and beautiful!
 
Last edited by a moderator:
  • #21
ThomasT said:
I think that you're wrong about this. So, unless you have some survey or poll of all physicists to back up your claim, then I guess that we can just disagree about this
Did you try the experiment I suggested of searching for "bell's theorem" plus "inequality" on google books? Like I said before, "would you disagree that if physics textbooks all present some theoretical conclusion as a fact, then it's a safe bet that this conclusion is one that the majority of physicists would agree with?"

And again, do a little self-reflection, do you really believe that as an amateur who has made no formal study of physics, you really have a rational basis for being confident that I (and DrChinese and RUTA and others) are wrong about the typical beliefs of physicists? You have cited only two papers with significant overlap in authorship, do you think that's enough to justify the conclusion that a significant fraction of the physics community disagrees with Bell's conclusions? Given how strongly you seem to want Bell's theorem to be wrong, do you think you are immune to effects like confirmation bias, not to mention the Dunning-Kruger effect?
ThomasT said:
Why are you even asking this? This is what I mean by obfuscation.
Please stop leaping to such mean-spirited conclusions, my questions are completely genuine.
ThomasT said:
Obviously, if you have a source emitting randomly polarized photons, then if you place a polarizer between the emitter and the detector, then the coeffiicient of transmission with the polarizer is 1/2 the photon flux without the polarizer.
To help explain my confusion, note that we aren't talking about the photon flux at an individual source, we're talking about the coincidence rate for two detectors at different locations A and B--you said "And the coincidence rate, ie. F(AB), is the same as with the original setup, .5 cos^2 |a-b|, in the ideal." In the "original setup" of the Bell test I was familiar with, when the polarizers were set to the same angle, then if there was a detection at A there'd be a probability of 1 that you'd also see a detection at B. I thought that would mean a "coincidence rate" of 1 as well. However, in the past I've normally only looked at the theoretical side of Bell's theorem and not the experimental side, so it seems like I've actually misunderstood what the "coincidence rate" equations are giving us. Looking at http://www.physics.princeton.edu/~mcdonald/.../QM/aspect_prl_47_460_81.pdf [Broken], the author writes on p. 3:
R(a,b) is the rate of coincidences with polarizer I in orientation a and polarizer II in orientation b ... and R0 is the coincidence rate with the two polarizers removed.
From a theoretical perspective it may be natural to think of coincidences just in terms of the probability that both members of the pair will respond the same way to the polarizers (which is how I was thinking of it, and some explanations of Bell's theorem do involve this sort of probability), but if they give R0 as the "coincidence rate with the two polarizers removed" that's presumably not what they're talking about since with no polarizers to block out the photons, efficient detectors would always detect both members of the pair with probability 1. Instead it seems that they are actually talking about a "rate" at which photons are being detected in time, like 6 photons/second or something. So in this case, the "coincidence rate" with polarizers at the same angle would naturally be half of R0, in spite of the fact that anytime an entangled photon makes it through polarizer A, its entangled twin is guaranteed to make it through polarizer B with probability 1. So if we want to know the coincidence rate expressed as a fraction of R0, the rate with no polarizers in place, it would indeed be 0.5 cos^2 (a-b).

So, sorry about the mistake, but there was really no need to accuse me of obfuscation since my confusion was genuine. And in my defense, I don't think there is perfect uniformity in how different authors use the phrase "coincidence rate", for example http://mist.npl.washington.edu/AV/altvw37.html says:
Suppose, for example, that we misalign the angles of the two polarization filters so that the angle between the polarization directions of the two filters is a=a1-a2. We measure the coincidence rate R(a), as compared to the rate Ro when the filters are perfectly aligned.

...

Quantum mechanics, on the other hand, predicts that the coincidence rate R(a1,a2) depends only on the relative angle a=a1-a2 between the two polarization directions, and that R(a) obeys Malus' Law. In other words, quantum mechanics predicts that R(a1,a2)=R(a)=RoCos2(a).
So here the equation does not include any term giving the coincidence rate when the filters are removed, in front of cos^2 it puts Ro, the rate when they are in place at the same angle. And presumably this would be a literal rate of photons/second, not a dimensionless 0.5.

Anyway, with the understanding that we're looking at the coincidence rate with polarizers at angles a and b expressed as a fraction of the rate of detection with no polarizers in place, I agree that your setup--where photons going towards A meet no polarizers, while photons going towards B encounter two polarizers at angles a and b in succession--will indeed yield the same coincidence rate as with the normal setup where the photon going towards B encounters a polarizer at angle b while the photon going towards A encounters a polarizer at angle a. However, the crucial point is that your setup does not meet the experimental conditions specified by Bell, which include the fact that there must be a spacelike separation between the random choice of angle a for the polarizer that photon #1 encounters and the event of photon #2 encountering the other polarizer at angle b. This spacelike separation is crucial, because in a local universe it means there can be no causal influence between the choice of angle a and the event of the photon either passing through or being reflected by b. This is why it would be quite possible to reproduce the 0.5 cos^2 (a-b) relationship you describe using classical optics (you'd just need a detector that gave a binary yes/no result depending on whether the light that reached it was above a certain threshold), but it would be impossible to reproduce the 0.5 cos^2 (a-b) relationship in a classical optics experiment that actually satisfied Bell's experimental conditions, including the part about the spacelike separation.

And this is why the following argument is just a giant strawman:
ThomasT said:
Here's what you've failed to address so far. Bell and Herbert say that the number of mismatches at 30 degrees (a-b) plus the number of mismatches at 30 degrees should be less than or equal to the number of mismatches at 60 degrees.
Yes, for the specific experimental setup described under the assumption of local realism, not for any arbitrary experimental setup that doesn't match the one they assumed.
ThomasT said:
But, according to Malus Law the number of mismatches at 60 degrees should be greater than the number of mismatches at 30 degrees + the number of mismatches at 30 degrees.
No, the classical Malus' law does not predict this, not in the setup that Bell and Herbert described. The fact that you can find a completely different setup where the classical Malus' law does predict mismatches at 60 is greater than the sum of mismatches at 30 is just a strawman argument, since Bell never argued that his inequalities should apply in any experiments other than ones meeting the conditions he specified. Since classical electromagnetism is a local realist theory, it would indeed be impossible to replicate the 0.5 cos^2 (a-b) in a classical optics experiment that actually matched Bell's setup.
 
Last edited by a moderator:
  • #22
OK, how about this one:


1. Take 2 classical sets of binary observables that are uncorrelated. Say, 2 stacks of 100 coins which were randomly flipped to H or T and then paired. One stack is Alice, the other is Bob. About 50 pairs will match (ideal case), about 50 will mismatch, so 0 net correlation.
2. Find a 3rd set which has this property: it is equally correlated to Alice and Bob (in the ideal case, ignore if off by 1). There should be such a set.
3. What is the correlation of that set to Alice and Bob? Classically, it MUST be exactly .5 (within some confidence level related to sample size). Because the midpoint of 100% matches and 50% matches is 75% matches, which is a correlation of 50% (75% matches less 25% mismatches).
4. However, in the quantum version of this (PDC Type I entangled photon pairs), the correlation is more like .7 (85% matches less 15% mismatches). Imagine Alice and Bob at 0 and 45 degrees. They will not be correlated at all. But their midpoint, 22.5 degrees, is 70% correlated (85% matched) to Alice AND 70% correlated (85% matched) to Bob. That's impossible with any classical set; ergo there is no classical set.

Or look at it from the other perspective:

1. I have 100 tossed coins (data values for a set at 22.5 degrees). Is it possible to obtain 2 different sets by turning over 15 coins from each set so that those new sets (Alice and Bob) are maximally different? Certainly, but how different can they be? Classically and quantum mechancially, you get different answers!
2. Classically, you will have sets that are no more than 30% different (15 changed in Alice + 15 changed in Bob out of 100 original).
3. In the QM world: When Alice is 0 degrees and Bob is 45 degrees (the farthest points where they are 15% different than the starting point of 22.5 degrees), well, those points are completely uncorrelated with each other (i.e. 50% different). Ergo, there are no classical datasets which have this simple observed property: 15%+15%=50%.
 
  • #23
DrChinese said:
OK, how about this one:


1. Take 2 classical sets of binary observables that are uncorrelated. Say, 2 stacks of 100 coins which were randomly flipped to H or T and then paired. One stack is Alice, the other is Bob. About 50 pairs will match (ideal case), about 50 will mismatch, so 0 net correlation.
2. Find a 3rd set which has this property: it is equally correlated to Alice and Bob (in the ideal case, ignore if off by 1). There should be such a set.
3. What is the correlation of that set to Alice and Bob? Classically, it MUST be exactly .5 (within some confidence level related to sample size). Because the midpoint of 100% matches and 50% matches is 75% matches, which is a correlation of 50% (75% matches less 25% mismatches).
4. However, in the quantum version of this (PDC Type I entangled photon pairs), the correlation is more like .7 (85% matches less 15% mismatches). Imagine Alice and Bob at 0 and 45 degrees. They will not be correlated at all. But their midpoint, 22.5 degrees, is 70% correlated (85% matched) to Alice AND 70% correlated (85% matched) to Bob. That's impossible with any classical set; ergo there is no classical set.

Or look at it from the other perspective:

1. I have 100 tossed coins (data values for a set at 22.5 degrees). Is it possible to obtain 2 different sets by turning over 15 coins from each set so that those new sets (Alice and Bob) are maximally different? Certainly, but how different can they be? Classically and quantum mechancially, you get different answers!
2. Classically, you will have sets that are no more than 30% different (15 changed in Alice + 15 changed in Bob out of 100 original).
3. In the QM world: When Alice is 0 degrees and Bob is 45 degrees (the farthest points where they are 15% different than the starting point of 22.5 degrees), well, those points are completely uncorrelated with each other (i.e. 50% different). Ergo, there are no classical datasets which have this simple observed property: 15%+15%=50%.
There's a difference in experiments involving coins and experiments involving light. Does that difference imply nonlocality? I don't think it does.
 
  • #24
JesseM, I didn't mean to imply that you've ever deliberately obfuscated anything. I probably should have phrased my directive as a request to you to please avoid unnecessarily complicating my little 'thought illustration' if at all possible. Apparently that wasn't possible. So, that's that. Anyway, it was just meant to illustrate a couple of things. Which I'll restate, but first the setup again:

The setup is an ideal optical Bell test setup where an ideal source is emitting pairs of counter-propagating photons, entangled in polarization, at a certain rate. There are two ideal photon detectors, A and B, placed opposite each other and equidistant from the emitter, registering detections at certain, identical, rates. Between the emitter and detectors are placed two polarizers, a and b. The detection rate at A with polarizer, a, in place is 1/2 the detection rate at A without polarizer, a, in place. The detection rate at A is invariant wrt rotations of the polarizer, a. The same holds for the B side. The rate of coincidental detection is .5cos^2 |a-b| .

Now, if eg. polarizer, a, is taken from the A side and transferred to the B side, then the rate of coincidental detection will still be .5cos^2 |a-b| . (The coincidence rate can't be greater than the maximum B side detection rate.)

I think that this idealization illustrates the following:

(1) It isn't just a 'coincidence', having nothing to do with optics, that the coincidence rate in the original setup is the same as the coincidence rate in the altered setup.
(2) There's clearly no need for 'nonlocal communication' in the altered setup (with a photon polariscope on the B side). So, I don't think we need nonlocality to understand the correlations in the original setup. (As if assuming 'nonlocality' is any sort of understanding anyway).
(3) Most importantly wrt the OP of this thread, either the altered setup is not fundamentally equivalent to the original setup, or Herbert and Bell are requiring light to behave in a way that contradicts at least two centuries of observational data (or, they're saying that Malus Law, as it applies to quantum polariscopic setups, is, per se, implying nonlocality).

Of course, one could maintain that the two setups are so different that in the original one (an idealization of the archetypal optical Bell test setup) with a polarizer on each side, some unknown field (or whatever) has been conjured from the depths of reality to enable the counter-propagating photons (or whatever) to communicate instantaneously (or, conveniently, as faster than light speed as the setup requires). But I find it difficult to imagine how the mere placement of a polarizer could have such, er, power, and think that a more plausible hypothesis is that people who think that violations of BI's (such as Herbert's 'simplest' BI) imply nonlocality have simply made a logical error somewhere along the line.

-----------------------

Wrt what most physicists would say, it's an open empirical question. My own sampling of the physics community has lead me to believe that most physicists would say that nature is evolving according to underlying dynamical principles in accordance with the principle of local action. But of course it wasn't a large sample, and the responses were maybe not representative of the responses one might get from thousands of physicists across all the sub-fields in physics. Some specialties might be more predisposed to liking certain ideas or thinking in certain terms (eg., that nature is local -- or nonlocal) than others.

-----------------------

I'm going to resist the temptation to reply to these most recent Bell threads for as long as it takes for me to catch up on my reading. I thank you, JesseM, for your detailed and sincere replies, and you and DrC and billschnieder and JenniT and my_wan and RUTA and several others for motivating me to learn more. I certainly agree that one must avoid cognitive bias and the dreaded D-K syndrome.

Maybe nature is nonlocal, or there are fields or media where disturbances propagate ftl. Who knows. These are open questions as far as I'm concerned. All I can say to those who think that Bell has definitively proved that nature is nonlocal is that I currently disagree with them, and, anyway, it doesn't seem to matter very much to physical science whether nature is fundamentally local or nonlocal -- because these aren't questions that can be answered scientifically.

To the OP, I think that Herbert's simplest Bell inequality is a valid Bell inequality, and that Herbert's interpretation of the physical meaning of its violation is not valid. But that's just my current opinion, and although that opinion is reinforced by a number of professional physicists (albeit probably not the majority of professional physicists), it might well change as my understanding of everything involved increases.

What I currently understand is the categorical logic involved in BIs, and the arithmetization thereof. What I don't currently understand is what this has to do with fundamental reality. Or, to phrase it differently, I don't understand why apparently many physicists think that BI violations imply anything about fundamental reality.

And, JesseM, here's a smiley for you. :smile: I was reading a recent reply of yours to RUTA in another thread. What you're saying there makes very good sense to me. That is, I agree with it. What you're saying in all of your replies, including those in this thread, makes sense to me. And, I think all your responses are well thought out and sincere. So, it's very difficult to disagree with you. However, I just don't happen to think, currently, that all of your statements, particularly pertaining to Bell, are necessarily correct. So, give me some time to read and think about this stuff and I'll get back to you -- and thanks again.

In the meantime, for interested readers, here are a couple of papers that you might agree are related to the Hess et al papers linked to correctly by JesseM in post #16.

Bell's inequalities I: An explanation for their experimental violation
Journal ref: Optics Communications 170 (1999) 55-60
http://arxiv.org/ftp/quant-ph/papers/0101/0101087.pdf

Bell's inequalities II: logical loophole in their interpretation
Journal ref: Optics Communications 170 (1999) 61-66
http://arxiv.org/ftp/quant-ph/papers/0101/0101094.pdf
 
Last edited:
  • #25
Loren Booda said:
Paraphrased from Nick Herbert's book Quantum Reality, the following (borrowed from the OP's http://quantumtantra.com/bell2.html" [Broken]) is the best (simplest) account I have found to date for the Bell inequality (SPOT stands for Single Photon Orientation Tester):

...

Just beautiful.

Yes, it’s very beautiful.

I don’t want to be a "party pooper"... but somehow I wonder if this is really the whole truth...
[Nick Herbert] What happened was that I ended up producing one of the simplest proofs of Nature's necessary deep non-locality.

...because John Bell himself used exactly the same example in a lecture in 1990...
N(+30°, -30°) ≤ N(+30°, 0°) + N(0°, -30°)

And this version is even shorter! :wink:
 
Last edited by a moderator:
  • #26
ThomasT said:
Wrt what most physicists would say, it's an open empirical question. My own sampling of the physics community has lead me to believe that most physicists would say that nature is evolving according to underlying dynamical principles in accordance with the principle of local action. But of course it wasn't a large sample, and the responses were maybe not representative of the responses one might get from thousands of physicists across all the sub-fields in physics. Some specialties might be more predisposed to liking certain ideas or thinking in certain terms (eg., that nature is local -- or nonlocal) than others.

I don’t think your 'sample' is representative:
RUTA said:
When I first entered the foundations community (1994), there were still a few conference presentations arguing that the statistical and/or experimental analyses of EPR-Bell experiments were flawed. Such talks have gone the way of the dinosaurs. Virtually everyone agrees that the EPR-Bell experiments and QM are legit, so we need a significant change in our worldview. There is a proper subset who believe this change will be related to the unification of QM and GR :-)

RUTA is a working PhD Professor of Physics.

ThomasT said:
I'm going to resist the temptation to reply to these most recent Bell threads for as long as it takes for me to catch up on my reading.
...
Maybe nature is nonlocal

ThomasT, I think it’s just great that you have found the word "Maybe"! I’m not a scientist, but I would guess that the M-word is the most useful of all in science!

Happy reading, and remember – no one is saying that it’s proved beyond any doubt that the world is nonlocal. We are just saying that Local Realism (LR) cannot reproduce all the predictions of QM and the results of all performed EPR-Bell experiments so far.

A little 'advice' on subjects to penetrate, while you’re reading, is the correlation on the relative angle between Alice & Bob:

Code:
[B]Angle	Alice	Bob[/B]
------------------------
0º	100%	100%
22.5º	85%	85%
45º	50%	50%
67.5º	15%	15%
90º	0%	0%

  • And don’t forget: Alice & Bob is outside each others lightcone when the final relative angle is set.

  • And don’t forget: We will not get this statistics on not entangled photons.

  • And don’t forget: This statistics would also be a fact if someone will be able to do EPR-Bell experiments with electrons instead of photons – Bell's Theorem is not restricted to optics.
Take care! :wink:
 
  • #27
DevilsAvocado said:
I don’t think your 'sample' is representative.
I have to agree. It was a sample of professional physicists, abeit none in the foundations field. So, pretty much not familiar with Bell stuff, and, as experimentalists, probably biased toward a 'local' worldview.

RUTA said:
When I first entered the foundations community (1994), there were still a few conference presentations arguing that the statistical and/or experimental analyses of EPR-Bell experiments were flawed. Such talks have gone the way of the dinosaurs. Virtually everyone agrees that the EPR-Bell experiments and QM are legit, so we need a significant change in our worldview. There is a proper subset who believe this change will be related to the unification of QM and GR :-)
DevilsAvocado said:
RUTA is a working PhD Professor of Physics.
The argument isn't that the experiments or the data analyses are flawed. So, I agree with RUTA on this. The argument is that the logic involved in assuming that violations of BIs allow inferences about fundamental reality is flawed. And so I don't agree with RUTA that the results of Bell tests require a significant change in our worldview.

DevilsAvocado said:
ThomasT, I think it’s just great that you have found the word "Maybe"! I’m not a scientist, but I would guess that the M-word is the most useful of all in science!
You don't have to be a professional scientist to be a scientist. I think you have a scientific mind. It's all in how you approach learning the truth of things. And, I agree that the willingness to say "maybe", as well as "I don't know" and "I don't understand" are very important prerequisites to learning.

But I like ruffling a few feathers now and then.

DevilsAvocado said:
Happy reading, and remember – no one is saying that it’s proved beyond any doubt that the world is nonlocal. We are just saying that Local Realism (LR) cannot reproduce all the predictions of QM and the results of all performed EPR-Bell experiments so far.
Well, then we pretty much agree.

DevilsAvocado said:
A little 'advice' on subjects to penetrate, while you’re reading, is the correlation on the relative angle between Alice & Bob:

Code:

Angle Alice Bob
------------------------
0º 100% 100%
22.5º 85% 85%
45º 50% 50%
67.5º 15% 15%
90º 0% 0%

* And don’t forget: Alice & Bob is outside each others lightcone when the final relative angle is set.
The loophole stuff is a red herring. The experiments are fine. It's all about the logic. The usual interpretation is a false dichotomy. But, that's just my current opinion, and might well be wrong. (I'm feeling very intellectually humble today for some reason. There will be no swindlin' today. Of course, tomorrow's another day.)

DevilsAvocado said:
* And don’t forget: We will not get this statistics on not entangled photons.
Of course. That's how the presence of entanglement is inferred -- from the statistical results.

DevilsAvocado said:
* And don’t forget: This statistics would also be a fact if someone will be able to do EPR-Bell experiments with electrons instead of photons – Bell's Theorem is not restricted to optics.
I'll definitely keep that in mind.

And this really is my last post on this stuff for a while (I didn't feel like reading this morning).
 
  • #28
ThomasT said:
I have to agree.
Great! :rolleyes:
ThomasT said:
So, I agree with RUTA on this.
Fantastic!
ThomasT said:
I think you have a scientific mind.
Thanks very much! :blushing:
ThomasT said:
But I like ruffling a few feathers now and then.
This is very reassuring... I thought you were ill there for a while...? (joke! :biggrin:)

(... I’m sharpening my teeth ... :rofl:)
ThomasT said:
Well, then we pretty much agree.
Cool! :cool:
ThomasT said:
The loophole stuff is a red herring.
Agree. :approve:
ThomasT said:
I'll definitely keep that in mind.
Very good, it will bring benefits. o:)
ThomasT said:
And this really is my last post on this stuff for a while
See you later, Terminator! :wink:
 
  • #29
ThomasT said:
Of course. That's how the presence of entanglement is inferred -- from the statistical results.

Entangled particles exist in a superposition of states. Classical particles do not exist in a superposition of states, they are in a single state. Classical of course meaning local realistic. So it seems pretty straightforward to me that finding particle pairs that have superposition statistics (i.e. Bell states) is an absolute counterexample to classical ideas. If, in fact, the particles were in mixed (i.e. classical) states, then you would get different statistics (Product state).

Again, the point is that entanglement (as a state) is incompatible with local realism.
 
  • #30
DrC, I’m looking for the stuff on "remote entanglement" (particles that never was in contact), but I can’t find it on PF?
 
  • #31
DevilsAvocado said:
DrC, I’m looking for the stuff on "remote entanglement" (particles that never was in contact), but I can’t find it on PF?

There are a batch of related ones from some of the top teams. Search on entanglement swapping, but here are a few:

http://arxiv.org/abs/quant-ph/0609135

We report for the first time in an ancilla-free process a non-local entanglement between two single photons which do not meet. For our experiment we derive a simple and efficient method to entangle two single photons using post-selection technology. The photons are guided into an interferometer setup without the need for ancilla photons for projection into the Bell-states. After passing the output ports, the photons are analyzed using a bell state analyzer on each side. The experimental data clearly shows a non-local interaction between these photons, surpassing the limit set by the CHSH-inequality with an S-value of 2.54 and 24 standard deviations.


http://arxiv.org/abs/0809.3991

Entanglement swapping allows to establish entanglement between independent particles that never interacted nor share any common past. This feature makes it an integral constituent of quantum repeaters. Here, we demonstrate entanglement swapping with time-synchronized independent sources with a fidelity high enough to violate a Clauser-Horne-Shimony-Holt inequality by more than four standard deviations. The fact that both entangled pairs are created by fully independent, only electronically connected sources ensures that this technique is suitable for future long-distance quantum communication experiments as well as for novel tests on the foundations of quantum physics.


http://arxiv.org/abs/quant-ph/0409093

We report the first experimental realization of entanglement swapping over large distances in optical fibers. Two photons separated by more than two km of optical fibers are entangled, although they never directly interacted. We use two pairs of time-bin entangled qubits created in spatially separated sources and carried by photons at telecommunication wavelengths. A partial Bell state measurement is performed with one photon from each pair which projects the two remaining photons, formerly independent onto an entangled state. A visibility high enough to violate a Bell inequality is reported, after both photons have each traveled through 1.1 km of optical fiber.


http://arxiv.org/abs/0911.1314

Quantum systems that have never interacted can become nonlocally correlated through a process called entanglement swapping. To characterize nonlocality in this context, we introduce local models where quantum systems that are initially uncorrelated are described by uncorrelated local variables. While a pair of maximally entangled qubits prepared in the usual way (i.e., emitted from a common source) requires a visibility close to 70% to violate a Bell inequality, we show that an entangled pair generated through entanglement swapping will already violate a Bell inequality for visibilities as low as 50% under our assumption.


---------------------------------

Although these experiments don't show it, you can entangle photons after they are detected... and you can even entangled photons that never existed at the same time.
 
  • #32
DrChinese said:
There are a batch of related ones from some of the top teams. Search on entanglement swapping, but here are a few:

BIG Thanks! :!)
 

Similar threads

Replies
50
Views
4K
Replies
80
Views
3K
Replies
1
Views
706
  • Quantum Physics
Replies
28
Views
1K
Replies
55
Views
6K
Replies
64
Views
4K
  • Quantum Interpretations and Foundations
10
Replies
333
Views
11K
  • Quantum Physics
2
Replies
47
Views
3K
Replies
50
Views
3K
  • Quantum Physics
3
Replies
95
Views
8K
Back
Top